Need help, to help, a friend with their HW (High School)

  • #1
Owen Ploe
4
2
Homework Statement
I am currently helping a friend with their physics HW as they are unable to do it. They are taking AP and I am not. So they have covered about 4 times as much material as I have.
Relevant Equations
ω = (ωo + ωf) / 2, L = I ω and others
Screen Shot 2024-04-24 at 7.29.57 PM.png

This is the problem set. I am stuck from this point... If anyone could give me a hand I would really appreciate it. I know this is probably really simple, but I don't know any of this and have been trying my best with youtube, and other peoples posts.
PS this is for high school
 
Physics news on Phys.org
  • #2
Owen Ploe said:
Homework Statement: I am currently helping a friend with their physics HW as they are unable to do it. They are taking AP and I am not. So they have covered about 4 times as much material as I have.
Relevant Equations: ω = (ωo + ωf) / 2, L = I ω and others

View attachment 344021
This is the problem set. I am stuck from this point... If anyone could give me a hand I would really appreciate it. I know this is probably really simple, but I don't know any of this and have been trying my best with youtube, and other peoples posts.
PS this is for high school
Using one of the relevant equations you quote, you should be able to fill in the last column of the first row easily. That gives you one way to calculate last column, last row.
Let the resulting rotation rate of the system be ##\omega## ( middle column, last row). That gives you another equation for last column, last row.
 
  • #3
haruspex said:
Using one of the relevant equations you quote, you should be able to fill in the last column of the first row easily. That gives you one way to calculate last column, last row.
Let the resulting rotation rate of the system be ##\omega## ( middle column, last row). That gives you another equation for last column, last row.
I got the first column. What I don't know is how to get the angular velocity of the student running, and then how I combined those two together.
Im sure this is really simple and kinda a stupid question. But thank you for helping me
 
  • #4
Owen Ploe said:
What I don't know is how to get the angular velocity of the student running
Then who wrote "2.79” under rad/s?
Owen Ploe said:
and then how I combined those two together
As I wrote, refer to the relevant equations in post #1. There are only two, and the other one is not relevant to this question.
Two more relevant equations:
Moment of Inertia of composite object about a given axis equals sum of MoIs of the individual components about that axis.
Angular Momentum of composite object about a given axis equals sum of AMs of the individual components about that axis.
 
  • #5
haruspex said:
Then who wrote "2.79” under rad/s?
My friend wrote the "2.79" and it was incorrect, the answers highlighted green are correct.
haruspex said:
Angular Momentum of composite object about a given axis equals sum of AMs of the individual components about that axis.
So basically, the angular momentum of the combined objects, is just AM of object one plus AM of object 2.
How do you find the AV of the student running? I know you can find it by AM/Rotational Inertia = AV
But I don't have the Rotational Inertia given to me.
 
  • #6
Owen Ploe said:
My friend wrote the "2.79" and it was incorrect
That's because linear velocity = radius x angular velocity. Your friend did angular velocity = radius x linear velocity. You can see that must be wrong by filling in the dimensions. Radius is a length, velocity is length/time, so radius x linear velocity would be length x length / time instead of 1/time.
Owen Ploe said:
the angular momentum of the combined objects, is just AM of object one plus AM of object 2.
Yes.
 
  • #7
haruspex said:
That's because linear velocity = radius x angular velocity. Your friend did angular velocity = radius x linear velocity. You can see that must be wrong by filling in the dimensions. Radius is a length, velocity is length/time, so radius x linear velocity would be length x length / time instead of 1/time.

Yes.
Thank you for all your help! I got all parts of the problem done. It was supper help full. Sorry some of my stuff was probably stupid.
 

Similar threads

  • Introductory Physics Homework Help
Replies
25
Views
2K
  • Introductory Physics Homework Help
Replies
3
Views
251
  • Introductory Physics Homework Help
Replies
2
Views
635
  • Introductory Physics Homework Help
Replies
5
Views
383
  • Introductory Physics Homework Help
Replies
24
Views
2K
Replies
28
Views
910
  • Introductory Physics Homework Help
Replies
4
Views
801
  • Introductory Physics Homework Help
Replies
30
Views
528
  • Introductory Physics Homework Help
Replies
6
Views
1K
  • Introductory Physics Homework Help
Replies
8
Views
2K
Back
Top